Mathematical induction proves that 1 h n 1 1 hn is true for any n in the range of 0 h 1.

Updated on educate 2024-02-08
8 answers
  1. Anonymous users2024-02-05

    Mathematical induction proves that (1-h) n 1 (1+hn) holds for any natural number n in the range of 0 h 1.

    Proof: n=1, 1-h) n=1-h

    1/(1+hn)=1/(1+h)

    by 0 h 1

    1-h 1 (1+h), 1-h 1, is clearly true.

    Assuming n=k, (1-h) k 1 (1+hk) holds.

    then n=k+1.

    1-h)^(k+1)=(1-h)^k(1-h)≤(1-h)/(1+hk)

    If (1-h) (1+hk) 1 [1+h(k+1)] 1) then (1-h)[1+h(k+1)] 1+hk1+h+hk -h-h -kh 1+hkh +kh 0

    by 0 h 1

    Since h 0, k > 0, the above equation holds, i.e., hypothesis (1) holds.

    According to the inductive method, (1-h) n 1 (1+hn) holds for any n in the range of 0 h 1.

  2. Anonymous users2024-02-04

    This is obviously true when n=1, and it is still true when n=k, i.e., (1-h) k<=1-kh

    When n=k+1, (1-h)(1-h) k<=(1-h)(1-kh)<(1-(k+1)h) is proved.

  3. Anonymous users2024-02-03

    n=2 ((n+1)/2)^n= [2+1)/2]^2= n!=2*1=2 So((n+1) 2) n> n!Establish.

    n>2 Suppose n=k holds the original formula, i.e., ((k+1) 2) k> k!i.e. (k+1) k 2 k>k!(1)

    Then n=k+1, ((k+1+1) 2) (k+1)=(k+2) (k+1) (2*2 k).2)

    Cause(k+2) (k+1)>2(k+1) (k+1).3)

    3) Substituting (2) (k+1+1) 2) (k+1)=(k+2) (k+1) (2*2 k)>2(k+1) (k+1) (2*2 k)=(k+1) (k+1) 2 k=(k+1)*(k+1) k 2 k(4)

    Substituting (1) into (4) gives ((k+1+1) 2) (k+1)>(k+1)*k!=(k+1)!i.e. n=k+1((n+1) 2) n > n!Establish.

  4. Anonymous users2024-02-02

    This is true when n=1.

    This is true when n=n.

    then when n=n+1.

    1+2+3+…+n+n+1

    1/2)n*(n+1)+n+1

    1/2)(n+2)*(n+1)

    Established, so established.

  5. Anonymous users2024-02-01

    Problem solving ideas: Directly use the mathematical induction method to prove the steps of the problem, and prove the inequality can be proved: (1) When n=2, [1 2+1+

    24] The proposition holds

    2) Suppose that when n=k, [1 k+1+

    k+2+k+3+…+

    2k 24] was founded.

    When n=k+1, [1 k+2+

    k+3+…+

    2k+2k+1+

    2k+2]=[1/k+1]+[1/k+2+k+3+…+

    2k+2k+1+

    2k+2]−

    k+1>2k+1+

    2k+2−k+1,∵[1/2k+1+

    2k+2−k+1=

    2(2k+1)(k+1)>0,∴

    k+1)+1+

    k+1)+2+

    k+1)+3+…+

    2(k+1)>

    24], when n=k+1 the proposition holds

    So for any n 2, n n* holds , 10,

  6. Anonymous users2024-01-31

    Proof 1: When n=1, the left side = 1*1!=1, right =(1+1)!-1=2-1=1

    i.e. left = right.

    2. The assumption that n=k(k 1) is the conclusion is true.

    i.e. 1 1!+2•2!+.k•k!=(k+1)!-1 then when n=k+1, 1 1!+2•2!+.k•k!+(k+1)(k+1)!

    k+1)!-1+(k+1)(k+1)!

    k+1)!+k+1)(k+1)!-1

    1+(k+1)](k+1)!-1

    k+2)(k+1)!-1

    k+2)!-1

    k+1+1)!-1

    That is, the conclusion of n=k+1 is true.

    Therefore, in summary, the original proposition is valid.

  7. Anonymous users2024-01-30

    When n=1, 1*(1+1)(2*1+1) 6=1, holds.

    When n=2, 2*(2+1)(2*2+1) 6=5, holds.

    Assuming n=k, 1 + 2 + 3 + ...n = n(n+1)(2n+1) 6 is true, but when n = k+1, 1 +2 +3 +....k²+(k+1)²=(1²+2²+3²+…k²)+k+1)²=k(k+1)(2k+1)/6+(k+1)²=(k+1)(k(2k+1)/6+(k+1))=(k+1)((2k²+k+6k+6)/6)=(k+1)(2k²+k+6k+6)/6

    k+1)(2k²+7k+6)/6

    k+1)((k+2)(2k+3))/6=(k+1)(k+2)(2k+3)/6

    k+1)((k+1)+1)(2(k+1)+1) 6.

  8. Anonymous users2024-01-29

    Proof that 1' when n=1, left = (2*1-1) =1, right = 1 3*1*(4*1 -1), left = right, the proposition is true.

    2' Suppose the proposition holds when n=k.

    i.e. 1 + 3 + 5 +2k-1) = 1 3k (4k-1) holds.

    Then when n=k+1, left=1 +3 +5+2k-1)²+2(k+1)-1]²=1/3k(4k²-1)+(2k+1)²

    1/3k+4k²+4k+1

    4/3k3+4k²+11/3k+1

    Right = 1 3 (k + 1) [4 (k + 1) -1] = (1 3k + 1 3) ( 4k +8k + 3) = 4 3k 3 + 8 3k + 8 3k + 1

    4/3k3+4k²+11/3k+1

    Left = Right, the proposition is true.

    The proposition from 'is that any n belongs to n*.

Related questions